La ricerca ha trovato 115 risultati

da Saro00
10 feb 2016, 18:36
Forum: Matematica ricreativa
Argomento: Somma periodica delle cifre
Risposte: 9
Visite : 11892

Re: Somma periodica delle cifre

Fatto 1: sia n in numero, n\equiv S_n \pmod{9} Dimostrazione: Sia n=a_0+a_1\cdot 10 + ... + a_k\cdot 10^k la sua scrittura in base 10 . Allora, n\equiv a_0+a_1\cdot 10 + ... + a_k\cdot 10^k \equiv a_0 + ...+ a_k \equiv ... \equiv S_n \pmod{9} dove la prima uguaglianza é giustificata dal fatto che 1...
da Saro00
24 gen 2016, 18:21
Forum: Algebra
Argomento: polinomio irriducibile
Risposte: 6
Visite : 4984

Re: polinomio irriducibile

Ciò che dice Lui (con la L Maiuscola) è che: 1. (x-a_1)^2\cdot ...\cdot(x-a_n)^2 + 1 non ha radici reali poichè è sempre maggiore di 1 . 2. Se per assurdo esistessero a_i tale che Q(a_i)=R(a_i)=-1 e a_j tale che Q(a_j)=R(a_j)=1 , allora per il teorema degli zeri (continuità dei polinomi) esiste una ...
da Saro00
24 gen 2016, 13:03
Forum: Algebra
Argomento: polinomio irriducibile
Risposte: 6
Visite : 4984

Re: polinomio irriducibile

Carino Se per assurdo la tesi fosse falsa, allora esisterebbero due polinomi Q(x) e R(x) a coefficienti interi tali che (x-a_1)^2\cdot ...\cdot (x-a_n)^2 + 1 = Q(x) \cdot R(x) e quest'ultima asserzione la chiamo s(x) Da s(a_i) ottengo che Q(a_i) \cdot R(a_i) = \pm 1 \forall i \in [1,2,...,n] \iff Q(...
da Saro00
21 gen 2016, 18:03
Forum: Teoria dei Numeri
Argomento: 194. Random
Risposte: 5
Visite : 3073

Re: 194. Random

Scusa per il ritardo.
Giusta, vai col prossimo.
da Saro00
16 gen 2016, 19:39
Forum: Teoria dei Numeri
Argomento: 194. Random
Risposte: 5
Visite : 3073

Re: 194. Random

In realtá no, oltre che la prima tua soluzione é sbagliata, ce ne sono altre
da Saro00
15 gen 2016, 14:53
Forum: Teoria dei Numeri
Argomento: 194. Random
Risposte: 5
Visite : 3073

194. Random

Determinare tutte le soluzioni $ (m,n) \in \mathbb{N^2} $ tali che
$ \displaystyle m^2+2\cdot 3^n=m\cdot (2^{n+1}-1) $
da Saro00
10 gen 2016, 16:56
Forum: Geometria
Argomento: Eulero Mente?
Risposte: 2
Visite : 2882

Eulero Mente?

Guardando un po' (2) di casi notevoli, ho notato questa cosa. Vorrei che qualche pro mi dicesse se é vera o meno (e in caso affermativo mi dicesse come si dimostra). Sia ABC un triangolo e P un punto tale che le rette di Eulero di ABP,\, ACP,\, BCP concorrono in Q . É sempre vero che Q sta sulla ret...
da Saro00
10 gen 2016, 16:45
Forum: Geometria
Argomento: Finalmente...
Risposte: 6
Visite : 3452

Re: Finalmente...

Giusta, proprio quella che intendevo.
Ti segnalo 2 typo.
$ N $ é la proiezione di $ D $ su $ AB $.
Quando usi Menelao, metti $ =-1 $.
Comunque era TST 2012 A1 Bosnia
da Saro00
09 gen 2016, 22:19
Forum: Geometria
Argomento: Finalmente...
Risposte: 6
Visite : 3452

Re: Finalmente...

Giusta :D
Metto in spoiler un idea per risolverlo in sintetica (e che spiega il titolo)
Testo nascosto:
Finalmente... Il teorema dei gialli
Testo nascosto:
I gialli di cui parlo non sono i cinesi
da Saro00
09 gen 2016, 16:48
Forum: Teoria dei Numeri
Argomento: 193. Numeri regolari
Risposte: 3
Visite : 3279

Re: 193. Numeri regolari

Appeno ho qualcosa di carino lo metto
da Saro00
09 gen 2016, 15:46
Forum: Teoria dei Numeri
Argomento: 193. Numeri regolari
Risposte: 3
Visite : 3279

Re: 193. Numeri regolari

Vediamo un po' Dimostro per induzione che esistono. Passo Base: (calato dal cielo, ma noo :D ) 3263442 = 2\cdot 3 \cdot 7 \cdot 13 \cdot 43 \cdot 139 è un numero 6 regolare. Infatti ha 6 divisori primi distinti e 1+1806+75894+466206+1087814+1631721=3263442 dove gli addendi dell'LHS sono tutti diviso...
da Saro00
09 gen 2016, 15:23
Forum: Geometria
Argomento: Finalmente...
Risposte: 6
Visite : 3452

Finalmente...

Sia $ ABC $ un triangolo ($ AB<AC $) e $ \Gamma $ la sua circoscritta.
Sia $ D $ il punto medio dell'arco $ BC $ contenente $ A $.
Sia $ E $ il piede della perpendicolare da $ D $ a $ AC $.
Dimostrare che $ \displaystyle CE=\frac{AB+AC}{2} $
da Saro00
09 gen 2016, 15:03
Forum: Geometria
Argomento: Invoco i Pro
Risposte: 4
Visite : 3425

Re: Invoco i Pro

Ed ecco la Meravigliosa soluzione di Sala. Innanzitutto definisco un po' di punti. B' il punto medio di ED e C' il punto medio di DF . Sia inoltre \Gamma ' la circonferenza di Feuerbach di DEF . Lemma 1: \Gamma e \omega_A hanno sempre almeno due punti in comune Dimostrazione: Considero il punto D , ...
da Saro00
08 gen 2016, 17:48
Forum: Olimpiadi della matematica
Argomento: Winter Camp 2016
Risposte: 134
Visite : 59282

Re: Winter Camp 2016

N-esima domanda.
Se uno non é spesato e ha chiesto di alloggiare con gli spesati, come funziona la prenotazione delle camere in albergo?
da Saro00
08 gen 2016, 17:33
Forum: Geometria
Argomento: Invoco i Pro
Risposte: 4
Visite : 3425

Re: Invoco i Pro

Grazie mille !! Inizio a scrivere la soluzione di dario2994, ma se riuscirò a trovare anche quella di Sala la scriverò. Innanzitutto definisco un po' di punti. X\equiv CB\cap FE , N il punto medio dell'arco BC non contenente A , S\equiv \Gamma \cap XN e L il punto medio di BC . Lemma 1: pow_{\Gamma}...